Skocz do zawartości

  •  
  • Mini kompendium
  • MimeTeX
  • Regulamin

Zdjęcie
        LICEUM        

Równanie w liczbach naturalnych



  • Nie możesz napisać tematu
  • Zaloguj się aby odpowiedzieć
8 odpowiedzi w tym temacie

#1 gustaw

gustaw

    Druga pochodna

  • Użytkownik
  • 117 postów
3
Neutralny
  • Płeć:Mężczyzna

Napisano 15.06.2014 - 22:23

Pokaż że równanie fc1ae2bcf14753490081fd57e35c496491e4fd10 nie ma rozwiązań w liczbach naturalnych.


  • 0

Afroman

    Kombinator

  • Użytkownik
3
  • Płeć:Kobieta

Napisano 25.09.2011 - 17:55

#2 Jarekzulus

Jarekzulus

    Wielki Analityk

  • +Mods
  • Redaktor
  • 4210 postów
3410
Profesor
  • Płeć:Mężczyzna

Napisano 15.06.2014 - 22:55

Może spróbuj tak:

 

a^2+b^2=d^2 zgodnie z tw. Pitagorasa

 

I teraz wystarczy udowodnić, że d^2+c^2=2012 nie ma rozwiązania dla liczb naturalnych a to łatwiejsze.

 

Brutalną siłą masz do sprawdzenia 14 pary liczb :) np. 45^2+1^2=2026 a więc za dużo sprawdzamy 44^2+9^2=2017 i tak dalej (podnoszę większą liczbę do kwadratu i doszacowuje drugą)

 

Rozwiązania brak więc ogólnie takiej trójki nie znajdziemy.

 

 

 

Można pobawić się kongruencjami ale na liceum to chyba za dużo


Użytkownik Jarekzulus edytował ten post 15.06.2014 - 22:56

  • 0

:wave: :wave: :wave: Jeśli rzuciłem choć promyczek światła na problem który postawiłeś - podziękuj. pre_1433974176__syg.jpgNad kreską


#3 gustaw

gustaw

    Druga pochodna

  • Użytkownik
  • 117 postów
3
Neutralny
  • Płeć:Mężczyzna

Napisano 16.06.2014 - 05:12

a jest jeszcze inna metoda?


  • 0

#4 Ereinion

Ereinion

    Mega Rozkminiacz z Marsa

  • $Jr Admin
  • 2104 postów
1008
Starszy Wykładowca I
  • Płeć:Mężczyzna

Napisano 16.06.2014 - 08:59

Nie można tak rozwiązywać tego zadania, choćby dlatego, że nie mamy żadnej gwarancji, że a i b to część pewnej trójki pitagorejskiej.

 

Poprawne rozwiązanie może wyglądać tak:

 

Zauważmy, że możliwe reszty kwadratowe modulo 4 to 0 i 1, skoro 4|2012, to musi być 4 | a^2, b^2, c^2. Czyli a,b,c są parzyste. Niech a = 2a_1 itd.

 

Mamy a_1^2 + b_1^2 + c_1^2 = 503. Dodatkowo 503 \equiv 7 \ \text{mod } 8 . Ale zbiór reszt kwadratowych modulo 8 to \{0, 1, 4\} . Sumując trzy liczby z tego zbioru nigdy nie dostaniemy 7. A to kończy dowód, że podane równanie nie ma rozwiązania.


Użytkownik Ereinion edytował ten post 16.06.2014 - 08:59

  • 2

#5 gustaw

gustaw

    Druga pochodna

  • Użytkownik
  • 117 postów
3
Neutralny
  • Płeć:Mężczyzna

Napisano 16.06.2014 - 10:08

w odpowiedziach jest napisane zeby uzyć dowodu niewprost


  • 0

#6 Ereinion

Ereinion

    Mega Rozkminiacz z Marsa

  • $Jr Admin
  • 2104 postów
1008
Starszy Wykładowca I
  • Płeć:Mężczyzna

Napisano 16.06.2014 - 10:45

To właśnie zrobiłem. Założyłem, że równanie ma rozwiązanie, powiedziałem coś o tym hipotetycznym rozwiązaniu i doszedłem do sprzeczności.


  • 1

#7 gustaw

gustaw

    Druga pochodna

  • Użytkownik
  • 117 postów
3
Neutralny
  • Płeć:Mężczyzna

Napisano 16.06.2014 - 11:17

ok dzięki


  • 0

#8 bb314

bb314

    miła suczka

  • $Jr Admin
  • Redaktor
  • 4000 postów
5064
Profesor
  • Płeć:Kobieta

Napisano 16.06.2014 - 14:07

Do tego można podejść również inną drogą, choć bardziej żmudną

 

suma trzech liczb jest parzysta, tzn. mamy dwie możliwości:

1. te trzy liczby są parzyste, lub

2. dwie liczby są nieparzyste i jedna parzysta

 

ad1.

a=2k\ \ \ b=2m\ \ \ c=2n

a^2+b^2+c^2=(2k)^2+(2m)^2+(2n)^2=4k^2+4m^2+4n^2=4(k^2+m^2+n^2)=2012\gr\ \Rightarrow\

\gr\ \Rightarrow\ k^2+m^2+n^2=503

suma trzech liczb jest nieparzysta, tzn. mamy dwie możliwości

1a. te trzy liczby są nieparzyste, lub

1b. jedna liczba jest nieparzysta i dwie parzyste

 

ad1a.

k=2p+1\ \ \ \ m=2q+1\ \ \ \ n=2r+1

k^2+m^2+n^2=(2p+1)^2+(2q+1)^2+(2r+1)^2=4p^2+4p+1+4q^2+4q+1+4r^2+4r+1=

=4(p^2+p+q^2+q+r^2+r)+3=503\gr\ \Rightarrow\ 4(p^2+p+q^2+q+r^2+r)=500\gr\ \Rightarrow\

\gr\ \Rightarrow\ p^2+p+q^2+q+r^2+r=125\gr\ \Rightarrow\ p(p+1)+q(q+1)+r(r+1)=125

po lewej stronie mamy sumę trzech składników parzystych (iloczyn dwóch kolejnych liczb naturalnych)

po prawej liczbę nieparzystą - sprzeczność

 
ad1b.
k=2p+1\ \ \ \ \ m=2q\ \ \ \ \ n=2r

k^2+m^2+n^2=(2p+1)^2+(2q)^2+(2r)^2=4p^2+4p+1+4q^2+4r^2=4(p^2+p+q^2+r^2)+1=503\gr\ \Rightarrow\

\gr\ \Rightarrow\ =4(p^2+p+q^2+q+r^2+r)=502

po lewej stronie mamy liczbę podzielną przez 4

po prawej stronie mamy liczbę niepodzielną przez cztery - sprzeczność

 

ad2.

a=2k+1\ \ \ b=2m+1\ \ \ c=2n

a^2+b^2+c^2=(2k+1)^2+(2m+1)^2+(2n)^2=4k^2+4k+1+4m^2+4m+1+4n^2=

=4(k^2+k+m^2+m+n^2)+2=2012\gr\ \Rightarrow\ 4(k^2+k+m^2+m+n^2)=2010

po lewej stronie mamy liczbę podzielną przez 4

po prawej stronie mamy liczbę niepodzielną przez cztery - sprzeczność

\ \ \ \ \ \ \ \ \ \ \ \ \ \ \ \ \ \ \ \ \ \ \ \ \ \ \ \ \ \ \ \ \ \ \ \ \ \ \ \ \ \ \ \ \ \ \ \ \ \ \ \ \ \ \ \ \ \ \ \ \ \ \ \ \ \ \ \ \ \ \ \ \ \ \ \ \ \ \ \ \ \ \ \ \ \ \ \ \ \ \ \ \ \ \ \ \ \ \ \ \ \ \ \ \ \ \ \ \ \ \ \ \ \ \ \ \ \ \ \ \ \ \ \ \ \ \ \ \ \ \ \ \ \ \ \ :shifty: \ :shifty:

  • 2

\ \
\ \ \ \ \ \ \ \ \ \ \ \ \ \ Jeśli chcesz powiedzieć \ \ DZIĘKUJĘ \ \ lub \ \ ŁAŁ \ \  to zaloguj się i kliknij znak\ rep_up.png\ nad kreską.\bl\ \ \ \nearrow
..
..
..
..
..
..


#9 Jarekzulus

Jarekzulus

    Wielki Analityk

  • +Mods
  • Redaktor
  • 4210 postów
3410
Profesor
  • Płeć:Mężczyzna

Napisano 16.06.2014 - 22:59

Nie można tak rozwiązywać tego zadania, choćby dlatego, że nie mamy żadnej gwarancji, że a i b to część pewnej trójki pitagorejskiej.

 

True :)


  • 0

:wave: :wave: :wave: Jeśli rzuciłem choć promyczek światła na problem który postawiłeś - podziękuj. pre_1433974176__syg.jpgNad kreską